Proving that a certain function is an integral of motion for a Hamiltonian












1












$begingroup$



Let $H=q_1p_1-q_2p_2-aq_1^2+bq_2^2$ (with $a,b$ constant) be a Hamiltionian.



Show that $G=dfrac{p_1-aq_1}{q_2}$ is a first integral (integral of motion) of this system.




According to the proposed solution, I need to show that the Poisson Bracket $[G,H] = 0$.



I used the following method, is this valid?



Other method?



The Hamiltionian equations are:



$$dot q_1 = dfrac{partial H}{partial p_1} = q_1 qquadqquad dot q_2 = -p_2$$



$$dot p_1 = -dfrac{partial H}{partial q_1} = -p_1+2aq_1 qquadqquad dot p_2 = q_2-2bq_2$$



Then, if $G$ is a integral of motion the total time derivative must equal zero (true?)



So:



$$begin{align}
dfrac{operatorname d G}{operatorname d t} & = dfrac{1}{q_2^2}left[ q_2(dot p_1-adot q_1) - dot q_2(p_1-aq_1)right]\
&= ldots text{(substitution using the Hamiltionian equations)} \
&= 0
end{align}$$










share|cite|improve this question









$endgroup$

















    1












    $begingroup$



    Let $H=q_1p_1-q_2p_2-aq_1^2+bq_2^2$ (with $a,b$ constant) be a Hamiltionian.



    Show that $G=dfrac{p_1-aq_1}{q_2}$ is a first integral (integral of motion) of this system.




    According to the proposed solution, I need to show that the Poisson Bracket $[G,H] = 0$.



    I used the following method, is this valid?



    Other method?



    The Hamiltionian equations are:



    $$dot q_1 = dfrac{partial H}{partial p_1} = q_1 qquadqquad dot q_2 = -p_2$$



    $$dot p_1 = -dfrac{partial H}{partial q_1} = -p_1+2aq_1 qquadqquad dot p_2 = q_2-2bq_2$$



    Then, if $G$ is a integral of motion the total time derivative must equal zero (true?)



    So:



    $$begin{align}
    dfrac{operatorname d G}{operatorname d t} & = dfrac{1}{q_2^2}left[ q_2(dot p_1-adot q_1) - dot q_2(p_1-aq_1)right]\
    &= ldots text{(substitution using the Hamiltionian equations)} \
    &= 0
    end{align}$$










    share|cite|improve this question









    $endgroup$















      1












      1








      1





      $begingroup$



      Let $H=q_1p_1-q_2p_2-aq_1^2+bq_2^2$ (with $a,b$ constant) be a Hamiltionian.



      Show that $G=dfrac{p_1-aq_1}{q_2}$ is a first integral (integral of motion) of this system.




      According to the proposed solution, I need to show that the Poisson Bracket $[G,H] = 0$.



      I used the following method, is this valid?



      Other method?



      The Hamiltionian equations are:



      $$dot q_1 = dfrac{partial H}{partial p_1} = q_1 qquadqquad dot q_2 = -p_2$$



      $$dot p_1 = -dfrac{partial H}{partial q_1} = -p_1+2aq_1 qquadqquad dot p_2 = q_2-2bq_2$$



      Then, if $G$ is a integral of motion the total time derivative must equal zero (true?)



      So:



      $$begin{align}
      dfrac{operatorname d G}{operatorname d t} & = dfrac{1}{q_2^2}left[ q_2(dot p_1-adot q_1) - dot q_2(p_1-aq_1)right]\
      &= ldots text{(substitution using the Hamiltionian equations)} \
      &= 0
      end{align}$$










      share|cite|improve this question









      $endgroup$





      Let $H=q_1p_1-q_2p_2-aq_1^2+bq_2^2$ (with $a,b$ constant) be a Hamiltionian.



      Show that $G=dfrac{p_1-aq_1}{q_2}$ is a first integral (integral of motion) of this system.




      According to the proposed solution, I need to show that the Poisson Bracket $[G,H] = 0$.



      I used the following method, is this valid?



      Other method?



      The Hamiltionian equations are:



      $$dot q_1 = dfrac{partial H}{partial p_1} = q_1 qquadqquad dot q_2 = -p_2$$



      $$dot p_1 = -dfrac{partial H}{partial q_1} = -p_1+2aq_1 qquadqquad dot p_2 = q_2-2bq_2$$



      Then, if $G$ is a integral of motion the total time derivative must equal zero (true?)



      So:



      $$begin{align}
      dfrac{operatorname d G}{operatorname d t} & = dfrac{1}{q_2^2}left[ q_2(dot p_1-adot q_1) - dot q_2(p_1-aq_1)right]\
      &= ldots text{(substitution using the Hamiltionian equations)} \
      &= 0
      end{align}$$







      solution-verification classical-mechanics






      share|cite|improve this question













      share|cite|improve this question











      share|cite|improve this question




      share|cite|improve this question










      asked May 15 '15 at 14:45









      dietervdfdietervdf

      2,6761327




      2,6761327






















          1 Answer
          1






          active

          oldest

          votes


















          0












          $begingroup$

          Your solution method and the proposed one are equivalent. Integrals of motion are those functions whose total time derivative is zero (by definition), and the total time derivative of a function $G$ dependent only on coordinates is equal to the Poisson bracket: $$begin{align*}dot{G} &= frac{dG}{dt}\ & = frac{partial G}{partial q_1}dot{q_1} + frac{partial G}{partial q_2}dot{q_2} + frac{partial G}{partial p_1}dot{p_1} + frac{partial G}{partial p_2}dot{p_2}\ &= frac{partial G}{partial q_1}frac{partial H}{partial p_1} + frac{partial G}{partial q_2}frac{partial H}{partial p_2} - frac{partial G}{partial p_1}frac{partial H}{partial q_1} - frac{partial G}{partial p_2}frac{partial H}{partial q_2}\ &=bigg(frac{partial G}{partial q_1}frac{partial H}{partial p_1} - frac{partial G}{partial p_1}frac{partial H}{partial q_1}bigg) + bigg(frac{partial G}{partial q_2}frac{partial H}{partial p_2} - frac{partial G}{partial p_2}frac{partial H}{partial q_2}bigg) \& = {G,H}end{align*}$$






          share|cite|improve this answer









          $endgroup$













            Your Answer





            StackExchange.ifUsing("editor", function () {
            return StackExchange.using("mathjaxEditing", function () {
            StackExchange.MarkdownEditor.creationCallbacks.add(function (editor, postfix) {
            StackExchange.mathjaxEditing.prepareWmdForMathJax(editor, postfix, [["$", "$"], ["\\(","\\)"]]);
            });
            });
            }, "mathjax-editing");

            StackExchange.ready(function() {
            var channelOptions = {
            tags: "".split(" "),
            id: "69"
            };
            initTagRenderer("".split(" "), "".split(" "), channelOptions);

            StackExchange.using("externalEditor", function() {
            // Have to fire editor after snippets, if snippets enabled
            if (StackExchange.settings.snippets.snippetsEnabled) {
            StackExchange.using("snippets", function() {
            createEditor();
            });
            }
            else {
            createEditor();
            }
            });

            function createEditor() {
            StackExchange.prepareEditor({
            heartbeatType: 'answer',
            autoActivateHeartbeat: false,
            convertImagesToLinks: true,
            noModals: true,
            showLowRepImageUploadWarning: true,
            reputationToPostImages: 10,
            bindNavPrevention: true,
            postfix: "",
            imageUploader: {
            brandingHtml: "Powered by u003ca class="icon-imgur-white" href="https://imgur.com/"u003eu003c/au003e",
            contentPolicyHtml: "User contributions licensed under u003ca href="https://creativecommons.org/licenses/by-sa/3.0/"u003ecc by-sa 3.0 with attribution requiredu003c/au003e u003ca href="https://stackoverflow.com/legal/content-policy"u003e(content policy)u003c/au003e",
            allowUrls: true
            },
            noCode: true, onDemand: true,
            discardSelector: ".discard-answer"
            ,immediatelyShowMarkdownHelp:true
            });


            }
            });














            draft saved

            draft discarded


















            StackExchange.ready(
            function () {
            StackExchange.openid.initPostLogin('.new-post-login', 'https%3a%2f%2fmath.stackexchange.com%2fquestions%2f1283580%2fproving-that-a-certain-function-is-an-integral-of-motion-for-a-hamiltonian%23new-answer', 'question_page');
            }
            );

            Post as a guest















            Required, but never shown

























            1 Answer
            1






            active

            oldest

            votes








            1 Answer
            1






            active

            oldest

            votes









            active

            oldest

            votes






            active

            oldest

            votes









            0












            $begingroup$

            Your solution method and the proposed one are equivalent. Integrals of motion are those functions whose total time derivative is zero (by definition), and the total time derivative of a function $G$ dependent only on coordinates is equal to the Poisson bracket: $$begin{align*}dot{G} &= frac{dG}{dt}\ & = frac{partial G}{partial q_1}dot{q_1} + frac{partial G}{partial q_2}dot{q_2} + frac{partial G}{partial p_1}dot{p_1} + frac{partial G}{partial p_2}dot{p_2}\ &= frac{partial G}{partial q_1}frac{partial H}{partial p_1} + frac{partial G}{partial q_2}frac{partial H}{partial p_2} - frac{partial G}{partial p_1}frac{partial H}{partial q_1} - frac{partial G}{partial p_2}frac{partial H}{partial q_2}\ &=bigg(frac{partial G}{partial q_1}frac{partial H}{partial p_1} - frac{partial G}{partial p_1}frac{partial H}{partial q_1}bigg) + bigg(frac{partial G}{partial q_2}frac{partial H}{partial p_2} - frac{partial G}{partial p_2}frac{partial H}{partial q_2}bigg) \& = {G,H}end{align*}$$






            share|cite|improve this answer









            $endgroup$


















              0












              $begingroup$

              Your solution method and the proposed one are equivalent. Integrals of motion are those functions whose total time derivative is zero (by definition), and the total time derivative of a function $G$ dependent only on coordinates is equal to the Poisson bracket: $$begin{align*}dot{G} &= frac{dG}{dt}\ & = frac{partial G}{partial q_1}dot{q_1} + frac{partial G}{partial q_2}dot{q_2} + frac{partial G}{partial p_1}dot{p_1} + frac{partial G}{partial p_2}dot{p_2}\ &= frac{partial G}{partial q_1}frac{partial H}{partial p_1} + frac{partial G}{partial q_2}frac{partial H}{partial p_2} - frac{partial G}{partial p_1}frac{partial H}{partial q_1} - frac{partial G}{partial p_2}frac{partial H}{partial q_2}\ &=bigg(frac{partial G}{partial q_1}frac{partial H}{partial p_1} - frac{partial G}{partial p_1}frac{partial H}{partial q_1}bigg) + bigg(frac{partial G}{partial q_2}frac{partial H}{partial p_2} - frac{partial G}{partial p_2}frac{partial H}{partial q_2}bigg) \& = {G,H}end{align*}$$






              share|cite|improve this answer









              $endgroup$
















                0












                0








                0





                $begingroup$

                Your solution method and the proposed one are equivalent. Integrals of motion are those functions whose total time derivative is zero (by definition), and the total time derivative of a function $G$ dependent only on coordinates is equal to the Poisson bracket: $$begin{align*}dot{G} &= frac{dG}{dt}\ & = frac{partial G}{partial q_1}dot{q_1} + frac{partial G}{partial q_2}dot{q_2} + frac{partial G}{partial p_1}dot{p_1} + frac{partial G}{partial p_2}dot{p_2}\ &= frac{partial G}{partial q_1}frac{partial H}{partial p_1} + frac{partial G}{partial q_2}frac{partial H}{partial p_2} - frac{partial G}{partial p_1}frac{partial H}{partial q_1} - frac{partial G}{partial p_2}frac{partial H}{partial q_2}\ &=bigg(frac{partial G}{partial q_1}frac{partial H}{partial p_1} - frac{partial G}{partial p_1}frac{partial H}{partial q_1}bigg) + bigg(frac{partial G}{partial q_2}frac{partial H}{partial p_2} - frac{partial G}{partial p_2}frac{partial H}{partial q_2}bigg) \& = {G,H}end{align*}$$






                share|cite|improve this answer









                $endgroup$



                Your solution method and the proposed one are equivalent. Integrals of motion are those functions whose total time derivative is zero (by definition), and the total time derivative of a function $G$ dependent only on coordinates is equal to the Poisson bracket: $$begin{align*}dot{G} &= frac{dG}{dt}\ & = frac{partial G}{partial q_1}dot{q_1} + frac{partial G}{partial q_2}dot{q_2} + frac{partial G}{partial p_1}dot{p_1} + frac{partial G}{partial p_2}dot{p_2}\ &= frac{partial G}{partial q_1}frac{partial H}{partial p_1} + frac{partial G}{partial q_2}frac{partial H}{partial p_2} - frac{partial G}{partial p_1}frac{partial H}{partial q_1} - frac{partial G}{partial p_2}frac{partial H}{partial q_2}\ &=bigg(frac{partial G}{partial q_1}frac{partial H}{partial p_1} - frac{partial G}{partial p_1}frac{partial H}{partial q_1}bigg) + bigg(frac{partial G}{partial q_2}frac{partial H}{partial p_2} - frac{partial G}{partial p_2}frac{partial H}{partial q_2}bigg) \& = {G,H}end{align*}$$







                share|cite|improve this answer












                share|cite|improve this answer



                share|cite|improve this answer










                answered Jan 22 at 20:03









                Ricardo BuringRicardo Buring

                1,4711334




                1,4711334






























                    draft saved

                    draft discarded




















































                    Thanks for contributing an answer to Mathematics Stack Exchange!


                    • Please be sure to answer the question. Provide details and share your research!

                    But avoid



                    • Asking for help, clarification, or responding to other answers.

                    • Making statements based on opinion; back them up with references or personal experience.


                    Use MathJax to format equations. MathJax reference.


                    To learn more, see our tips on writing great answers.




                    draft saved


                    draft discarded














                    StackExchange.ready(
                    function () {
                    StackExchange.openid.initPostLogin('.new-post-login', 'https%3a%2f%2fmath.stackexchange.com%2fquestions%2f1283580%2fproving-that-a-certain-function-is-an-integral-of-motion-for-a-hamiltonian%23new-answer', 'question_page');
                    }
                    );

                    Post as a guest















                    Required, but never shown





















































                    Required, but never shown














                    Required, but never shown












                    Required, but never shown







                    Required, but never shown

































                    Required, but never shown














                    Required, but never shown












                    Required, but never shown







                    Required, but never shown







                    Popular posts from this blog

                    Can a sorcerer learn a 5th-level spell early by creating spell slots using the Font of Magic feature?

                    Does disintegrating a polymorphed enemy still kill it after the 2018 errata?

                    A Topological Invariant for $pi_3(U(n))$